N has all divisors up to 31 except two The Next CEO of Stack OverflowNumber theory problem divides“Quadly” numbers with just 4 factorsTrue or false division algorithm problemProve that $1$ has only one divisorA Question Related to Zsigmondy's Theoremnumber system and divisibilityWritten as sum of positive divisorsUnder what conditions does an integer dividing the square of n imply that the integer must divide n?divisibility proofProve that if $3n^2 + 2n$ is even, then $n$ is even

Is there an equivalent of cd - for cp or mv

Purpose of level-shifter with same in and out voltages

How do you define an element with an ID attribute using LWC?

How to Implement Deterministic Encryption Safely in .NET

What CSS properties can the br tag have?

Would a grinding machine be a simple and workable propulsion system for an interplanetary spacecraft?

Free fall ellipse or parabola?

Aggressive Under-Indexing and no data for missing index

Man transported from Alternate World into ours by a Neutrino Detector

How to avoid supervisors with prejudiced views?

What flight has the highest ratio of timezone difference to flight time?

Reshaping json / reparing json inside shell script (remove trailing comma)

Is it okay to majorly distort historical facts while writing a fiction story?

Getting Stale Gas Out of a Gas Tank w/out Dropping the Tank

In the "Harry Potter and the Order of the Phoenix" video game, what potion is used to sabotage Umbridge's speakers?

What is the process for purifying your home if you believe it may have been previously used for pagan worship?

Why do we say 'Un seul M' and not 'Une seule M' even though M is a "consonne"

Won the lottery - how do I keep the money?

Cannot shrink btrfs filesystem although there is still data and metadata space left : ERROR: unable to resize '/home': No space left on device

Is Nisuin Biblical or Rabbinic?

Could a dragon use its wings to swim?

Airplane gently rocking its wings during whole flight

Help/tips for a first time writer?

Which one is the true statement?



N has all divisors up to 31 except two



The Next CEO of Stack OverflowNumber theory problem divides“Quadly” numbers with just 4 factorsTrue or false division algorithm problemProve that $1$ has only one divisorA Question Related to Zsigmondy's Theoremnumber system and divisibilityWritten as sum of positive divisorsUnder what conditions does an integer dividing the square of n imply that the integer must divide n?divisibility proofProve that if $3n^2 + 2n$ is even, then $n$ is even










0












$begingroup$


Let $N$ be a positive integer. It is true that $1|N, 2|N, 3|N, dots, 31|N$, except two. Which are false?



It seems like the problem is impossible. For example, any two consecutive integers must have an even integer E, and this even integer is divisible by 2 and an integer less than it, and since 2 and that integer divide N, E must divide $N$, which is a contradiction. For example, if we chose 31 and 30, then since 2 and 15 divide N, it must be the case that 30 divides $N$, which is a contradiction. So I'm not sure where to go with this problem.










share|cite|improve this question









$endgroup$











  • $begingroup$
    I understand the problem as follows: Of the 31 statements $i|N, i=1,2,ldots,31$ 29 are true and 2 are false. Which are the 2 that are false? As your example shows, an even divisor is likely not among them.
    $endgroup$
    – Ingix
    Mar 19 at 21:42










  • $begingroup$
    It seems like there is more than one solution. We could have $$N= 31!over 17cdot19$$ but we could choose any two primes $17leq p,qleq31$ in the denominator.
    $endgroup$
    – saulspatz
    Mar 19 at 22:10















0












$begingroup$


Let $N$ be a positive integer. It is true that $1|N, 2|N, 3|N, dots, 31|N$, except two. Which are false?



It seems like the problem is impossible. For example, any two consecutive integers must have an even integer E, and this even integer is divisible by 2 and an integer less than it, and since 2 and that integer divide N, E must divide $N$, which is a contradiction. For example, if we chose 31 and 30, then since 2 and 15 divide N, it must be the case that 30 divides $N$, which is a contradiction. So I'm not sure where to go with this problem.










share|cite|improve this question









$endgroup$











  • $begingroup$
    I understand the problem as follows: Of the 31 statements $i|N, i=1,2,ldots,31$ 29 are true and 2 are false. Which are the 2 that are false? As your example shows, an even divisor is likely not among them.
    $endgroup$
    – Ingix
    Mar 19 at 21:42










  • $begingroup$
    It seems like there is more than one solution. We could have $$N= 31!over 17cdot19$$ but we could choose any two primes $17leq p,qleq31$ in the denominator.
    $endgroup$
    – saulspatz
    Mar 19 at 22:10













0












0








0





$begingroup$


Let $N$ be a positive integer. It is true that $1|N, 2|N, 3|N, dots, 31|N$, except two. Which are false?



It seems like the problem is impossible. For example, any two consecutive integers must have an even integer E, and this even integer is divisible by 2 and an integer less than it, and since 2 and that integer divide N, E must divide $N$, which is a contradiction. For example, if we chose 31 and 30, then since 2 and 15 divide N, it must be the case that 30 divides $N$, which is a contradiction. So I'm not sure where to go with this problem.










share|cite|improve this question









$endgroup$




Let $N$ be a positive integer. It is true that $1|N, 2|N, 3|N, dots, 31|N$, except two. Which are false?



It seems like the problem is impossible. For example, any two consecutive integers must have an even integer E, and this even integer is divisible by 2 and an integer less than it, and since 2 and that integer divide N, E must divide $N$, which is a contradiction. For example, if we chose 31 and 30, then since 2 and 15 divide N, it must be the case that 30 divides $N$, which is a contradiction. So I'm not sure where to go with this problem.







divisibility






share|cite|improve this question













share|cite|improve this question











share|cite|improve this question




share|cite|improve this question










asked Mar 19 at 21:28









WesleyWesley

609413




609413











  • $begingroup$
    I understand the problem as follows: Of the 31 statements $i|N, i=1,2,ldots,31$ 29 are true and 2 are false. Which are the 2 that are false? As your example shows, an even divisor is likely not among them.
    $endgroup$
    – Ingix
    Mar 19 at 21:42










  • $begingroup$
    It seems like there is more than one solution. We could have $$N= 31!over 17cdot19$$ but we could choose any two primes $17leq p,qleq31$ in the denominator.
    $endgroup$
    – saulspatz
    Mar 19 at 22:10
















  • $begingroup$
    I understand the problem as follows: Of the 31 statements $i|N, i=1,2,ldots,31$ 29 are true and 2 are false. Which are the 2 that are false? As your example shows, an even divisor is likely not among them.
    $endgroup$
    – Ingix
    Mar 19 at 21:42










  • $begingroup$
    It seems like there is more than one solution. We could have $$N= 31!over 17cdot19$$ but we could choose any two primes $17leq p,qleq31$ in the denominator.
    $endgroup$
    – saulspatz
    Mar 19 at 22:10















$begingroup$
I understand the problem as follows: Of the 31 statements $i|N, i=1,2,ldots,31$ 29 are true and 2 are false. Which are the 2 that are false? As your example shows, an even divisor is likely not among them.
$endgroup$
– Ingix
Mar 19 at 21:42




$begingroup$
I understand the problem as follows: Of the 31 statements $i|N, i=1,2,ldots,31$ 29 are true and 2 are false. Which are the 2 that are false? As your example shows, an even divisor is likely not among them.
$endgroup$
– Ingix
Mar 19 at 21:42












$begingroup$
It seems like there is more than one solution. We could have $$N= 31!over 17cdot19$$ but we could choose any two primes $17leq p,qleq31$ in the denominator.
$endgroup$
– saulspatz
Mar 19 at 22:10




$begingroup$
It seems like there is more than one solution. We could have $$N= 31!over 17cdot19$$ but we could choose any two primes $17leq p,qleq31$ in the denominator.
$endgroup$
– saulspatz
Mar 19 at 22:10










0






active

oldest

votes












Your Answer





StackExchange.ifUsing("editor", function ()
return StackExchange.using("mathjaxEditing", function ()
StackExchange.MarkdownEditor.creationCallbacks.add(function (editor, postfix)
StackExchange.mathjaxEditing.prepareWmdForMathJax(editor, postfix, [["$", "$"], ["\\(","\\)"]]);
);
);
, "mathjax-editing");

StackExchange.ready(function()
var channelOptions =
tags: "".split(" "),
id: "69"
;
initTagRenderer("".split(" "), "".split(" "), channelOptions);

StackExchange.using("externalEditor", function()
// Have to fire editor after snippets, if snippets enabled
if (StackExchange.settings.snippets.snippetsEnabled)
StackExchange.using("snippets", function()
createEditor();
);

else
createEditor();

);

function createEditor()
StackExchange.prepareEditor(
heartbeatType: 'answer',
autoActivateHeartbeat: false,
convertImagesToLinks: true,
noModals: true,
showLowRepImageUploadWarning: true,
reputationToPostImages: 10,
bindNavPrevention: true,
postfix: "",
imageUploader:
brandingHtml: "Powered by u003ca class="icon-imgur-white" href="https://imgur.com/"u003eu003c/au003e",
contentPolicyHtml: "User contributions licensed under u003ca href="https://creativecommons.org/licenses/by-sa/3.0/"u003ecc by-sa 3.0 with attribution requiredu003c/au003e u003ca href="https://stackoverflow.com/legal/content-policy"u003e(content policy)u003c/au003e",
allowUrls: true
,
noCode: true, onDemand: true,
discardSelector: ".discard-answer"
,immediatelyShowMarkdownHelp:true
);



);













draft saved

draft discarded


















StackExchange.ready(
function ()
StackExchange.openid.initPostLogin('.new-post-login', 'https%3a%2f%2fmath.stackexchange.com%2fquestions%2f3154637%2fn-has-all-divisors-up-to-31-except-two%23new-answer', 'question_page');

);

Post as a guest















Required, but never shown

























0






active

oldest

votes








0






active

oldest

votes









active

oldest

votes






active

oldest

votes















draft saved

draft discarded
















































Thanks for contributing an answer to Mathematics Stack Exchange!


  • Please be sure to answer the question. Provide details and share your research!

But avoid


  • Asking for help, clarification, or responding to other answers.

  • Making statements based on opinion; back them up with references or personal experience.

Use MathJax to format equations. MathJax reference.


To learn more, see our tips on writing great answers.




draft saved


draft discarded














StackExchange.ready(
function ()
StackExchange.openid.initPostLogin('.new-post-login', 'https%3a%2f%2fmath.stackexchange.com%2fquestions%2f3154637%2fn-has-all-divisors-up-to-31-except-two%23new-answer', 'question_page');

);

Post as a guest















Required, but never shown





















































Required, but never shown














Required, but never shown












Required, but never shown







Required, but never shown

































Required, but never shown














Required, but never shown












Required, but never shown







Required, but never shown







Popular posts from this blog

Lowndes Grove History Architecture References Navigation menu32°48′6″N 79°57′58″W / 32.80167°N 79.96611°W / 32.80167; -79.9661132°48′6″N 79°57′58″W / 32.80167°N 79.96611°W / 32.80167; -79.9661178002500"National Register Information System"Historic houses of South Carolina"Lowndes Grove""+32° 48' 6.00", −79° 57' 58.00""Lowndes Grove, Charleston County (260 St. Margaret St., Charleston)""Lowndes Grove"The Charleston ExpositionIt Happened in South Carolina"Lowndes Grove (House), Saint Margaret Street & Sixth Avenue, Charleston, Charleston County, SC(Photographs)"Plantations of the Carolina Low Countrye

random experiment with two different functions on unit interval Announcing the arrival of Valued Associate #679: Cesar Manara Planned maintenance scheduled April 23, 2019 at 00:00UTC (8:00pm US/Eastern)Random variable and probability space notionsRandom Walk with EdgesFinding functions where the increase over a random interval is Poisson distributedNumber of days until dayCan an observed event in fact be of zero probability?Unit random processmodels of coins and uniform distributionHow to get the number of successes given $n$ trials , probability $P$ and a random variable $X$Absorbing Markov chain in a computer. Is “almost every” turned into always convergence in computer executions?Stopped random walk is not uniformly integrable

How should I support this large drywall patch? Planned maintenance scheduled April 23, 2019 at 00:00UTC (8:00pm US/Eastern) Announcing the arrival of Valued Associate #679: Cesar Manara Unicorn Meta Zoo #1: Why another podcast?How do I cover large gaps in drywall?How do I keep drywall around a patch from crumbling?Can I glue a second layer of drywall?How to patch long strip on drywall?Large drywall patch: how to avoid bulging seams?Drywall Mesh Patch vs. Bulge? To remove or not to remove?How to fix this drywall job?Prep drywall before backsplashWhat's the best way to fix this horrible drywall patch job?Drywall patching using 3M Patch Plus Primer